Bạn chưa đăng nhập. Vui lòng đăng nhập để hỏi bài

Những câu hỏi liên quan
tinh nguyenduc
Xem chi tiết
Vũ Quang Huy
2 tháng 5 2022 lúc 15:27

tham khảo

Đặt A = 1/3 + 1/9 + 1/27 + 1/81 + 1/243 + 1/729 
A x 3 = 3 x (1/3 + 1/9 + 1/27 + 1/81 + 1/243 + 1/729) 
= 1 + 1/3 + 1/9 + 1/27 + 1/81 + 1/243 
A x 3 - A = 1 + 1/3 + 1/9 + 1/27 + 1/81 + 1/243 - (1/3 + 1/9 + 1/27 + 1/81 + 1/243 + 1/729) 
= 1 + 1/3 + 1/9 + 1/27 + 1/81 + 1/243 - 1/3 - 1/9 - 1/27 - 1/81 - 1/243 - 1/729 
= 1 - 1/729 
A x 2 = 728/729 
A = 364/729

Chuu
2 tháng 5 2022 lúc 15:34

\(\dfrac{243}{729}+\dfrac{81}{729}+\dfrac{27}{729}+\dfrac{9}{729}+\dfrac{2}{729}+\dfrac{1}{729}=\dfrac{243+81+27+9+2+1}{729}=\dfrac{363}{729}=\dfrac{121}{243}\)

★彡✿ทợท彡★
2 tháng 5 2022 lúc 15:35

 

 

Lê Ánh Diễm My
Xem chi tiết
Đỗ Hoàng Tâm Như
19 tháng 5 2022 lúc 19:11

Đặt A = 1/3 + 1/9 + 1/27 + 1/81 + 1/243 + 1/729 
A x 3 = 3 x (1/3 + 1/9 + 1/27 + 1/81 + 1/243 + 1/729) 
= 1 + 1/3 + 1/9 + 1/27 + 1/81 + 1/243 
A x 3 - A = 1 + 1/3 + 1/9 + 1/27 + 1/81 + 1/243 - (1/3 + 1/9 + 1/27 + 1/81 + 1/243 + 1/729) 
= 1 + 1/3 + 1/9 + 1/27 + 1/81 + 1/243 - 1/3 - 1/9 - 1/27 - 1/81 - 1/243 - 1/729 
= 1 - 1/729 
A x 2 = 728/729 
A = 364/729

Phan An An
Xem chi tiết
nguyễn thị hương giang
16 tháng 10 2023 lúc 19:39

\(3A=3\cdot\left(\dfrac{1}{3}+\dfrac{1}{9}+\dfrac{1}{27}+\dfrac{1}{81}+\dfrac{1}{243}+\dfrac{1}{729}\right)\)

\(\Rightarrow3A=1+\dfrac{1}{3}+\dfrac{1}{9}+\dfrac{1}{27}+\dfrac{1}{81}+\dfrac{1}{243}\)

Lấy \(3A-A=1-\dfrac{1}{729}\)

\(\Rightarrow2A=\dfrac{728}{729}\Rightarrow A=\dfrac{364}{729}\)

Lương Phú Cường
16 tháng 10 2023 lúc 19:36

A=1/3+1/9+1/27+1/81+1/243+1/729

3A=1+1/3+1/9+1/27+1/81+1/243

3A-A=(1+1/3+1/9+1/27+1/81+1/243)-(1/3+1/9+1/27+1/81+1/243+1/729)

3A-A=1-1/3+1/3-1/9+1/9-1/27+1/27-1/81+1/81-1/243+1/243-1/729)

2A=1-1/729

2A=728/729

A=728/729/2

A=364/729

Thank you
Xem chi tiết
Vương Thị Diễm Quỳnh
11 tháng 9 2016 lúc 8:19

đặt S=\(\frac{1}{3}+\frac{1}{9}+\frac{1}{27}+\frac{1}{81}+\frac{1}{243}+\frac{1}{729}\)

=>3S= \(1+\frac{1}{3}+\frac{1}{9}+\frac{1}{27}+\frac{1}{81}+\frac{1}{243}\)

=>3S-S=\(\left(1+\frac{1}{3}+...+\frac{1}{243}\right)-\left(\frac{1}{3}+\frac{1}{9}+...+\frac{1}{729}\right)\)

=>s=1-1/729 = 728/729

Tuong Minh Huy
11 tháng 9 2016 lúc 8:12

1/3+1/9+1/27+1/81+1/243+1/729=(1/3+1/9+1/81)+(1/27+1/243+1/729)=37/81+37/729=333/729+37/729=370/729

Thảo
11 tháng 9 2016 lúc 8:18

= 370/729

Điểm âm của mik chỉ còn 109 nữa thôi, các bn giúp mik nha

ai giúp mik, mik sẽ giúp lại

cảm ơn trc~~~

Nguyễn Huyền Trang
Xem chi tiết
Nguyễn  Linh Chi
Xem chi tiết
Mianca Sarana
23 tháng 4 2016 lúc 21:14

1 + 1/3 + 1/9 + 1/27 + 1/81 + 1/243 + 1/729

=1+ 243/729+ 81/729 + 27/729 + 9/729 + 3/729

=1093/729

Mianca Sarana
23 tháng 4 2016 lúc 21:07

1 + 1/3 + 1/9 + 1/27 + 1/81 + 1/243 + 1/729

=1093/729

Nguyễn  Linh Chi
23 tháng 4 2016 lúc 21:10

làm từng bước đi nhé bạn.kết quả mình cũng ra rồi nhưng mình muốn tham khảo cách làm của các bạn

Đoàn Thị Mai Hương
Xem chi tiết
Nguyễn Đình Dũng
3 tháng 5 2015 lúc 15:22

1 + 1/3 + 1/9+1/27+1/81+1/243+1/729

=1+1-1/3+1/3-1/9+1/9-1/27-1/27-1/81+1/81-1/243

= 2 - 1/243

=485/243

nobita
Xem chi tiết
Mianca Sarana
23 tháng 4 2016 lúc 21:05

1+ 1/3 + 1/9 + 1/27 + 1/81 + 1/243 + 1/729

=1093/729

nguyentancuong
23 tháng 4 2016 lúc 21:10

A=1+1/3+1/3^2+......1/3^6

3A= 3 +1 + 1/3+......=1/3^5

3A-A= 3-1/3^6

A=\(\frac{3-\frac{1}{3^6}}{2}\)

Phạm Huyền My
23 tháng 4 2016 lúc 21:24

\(A=1+\frac{1}{3}+\frac{1}{9}+\frac{1}{27}+\frac{1}{81}+\frac{1}{243}+\frac{1}{729}\)

\(A\)\(x\)\(3=3x\left(1+\frac{1}{3}+\frac{1}{9}+\frac{1}{27}+\frac{1}{81}+\frac{1}{243}+\frac{1}{729}\right)\)

\(A\)\(x3=3+1+\frac{1}{3}+\frac{1}{9}+\frac{1}{27}+\frac{1}{81}+\frac{1}{243}\)

\(A\)\(x3=1+\frac{1}{3}+\frac{1}{9}+\frac{1}{27}+\frac{1}{81}+\frac{1}{243}+3\)

\(-A=1+\frac{1}{3}+\frac{1}{9}+\frac{1}{27}+\frac{1}{81}+\frac{1}{243}+\frac{1}{729}\)

\(A\)\(x2=3-\frac{1}{729}=\frac{2186}{729}\)

\(A=\frac{2186}{729}:2=\frac{1093}{729}\)

linhda nhi
Xem chi tiết
thiên thần trái đất
12 tháng 5 2017 lúc 11:52

1093/729

anhduc1501
12 tháng 5 2017 lúc 11:56

\(A=1+\frac{1}{3}+\frac{1}{9}+\frac{1}{27}+\frac{1}{81}+\frac{1}{243}+\frac{1}{729}\)

\(3\times A=3+1+\frac{1}{3}+\frac{1}{9}+\frac{1}{27}+\frac{1}{81}+\frac{1}{243}\)

\(3\times A-A=3-\frac{1}{729}=\frac{2186}{729}\)

\(2\times A=\frac{2186}{729}=>A=\frac{1093}{729}\)